User Avatar
cpdevill307
Joined
Apr 2025
Subscription
Free

Let me preface this by saying I know I should be using official questions, but I bought this book in a rush for a trip where I wouldn't have great internet (powerscore books haven't arrived yet). My thoughts on the question at the bottom so as to not influence anyone before reading.

Despite five consecutive years in which global consumption of grain has been greater than global production, it is unlikely that the world is facing a near-term crisis in the food supply. The average shortfalls have been mainly due to reduced output from farms in China, which is moving from a policy of central control over agricultural production to a more market-driven model. Therefore, if demand for grain continues to fall short of supply, Chinese production of grain should increase dramatically. Which one of the following principles most helps to justify this reasoning?

A. Global markets respond more slowly than regional markets, so local rates of production usually change more rapidly than the global average.

B. When agricultural production is centrally controlled, it is unable to respond to changing demand by adjusting rates of supply.

C. Average shortfalls are most readily remedied by local increases in production.

D. When agricultural production is market-driven, it is likely to respond to rising demand by increasing production.

E. Centrally controlled agricultural production has been shown to be more inefficient than market-driven models.

.

Answer/Explanation below.

.

.

.

.

.

.

.

.

.

.

.

.

.

.

Explanation:

Correct Answer: D

When agricultural production is market-driven, it is likely to respond to rising demand by increasing production.

The speaker concludes that Chinese production will rise if demand requires it based on the premise that Chinese production is now market-driven. Clearly the author connects market-driven methods with matching supply and demand. Choice A isn't right because it isn't actually about the difference between global and regional markets but about China's transformation to a market economy. Choice B doesn't justify the conclusion but explains how things worked under China's older system. Choice C is wrong because the speaker isn't talking about local production. Choice D looks like the right answer. The speaker bases his conclusion on the principle that a market economy will respond to increased demand with increased production. Choice E isn't right. The speaker doesn't mention inefficiency or even compare a market-driven system to a centrally regulated one; he's only interested in the effects of China's transition between the two. Choice D is the best answer.

I think they swapped the words supply and demand in the conclusion and that it should read "Therefore, if supply for grain continues to fall short of demand, Chinese production of grain should increase dramatically." I know questions don't have to reflect reality, but this isn't even internally consistent. Continues should refer to the situation presented in the first sentence, but the first sentence clearly says that consumption (demand) is greater than production (supply). And the first line of the explanation also clearly says "The speaker concludes that Chinese production will rise if demand requires it", and demand would only require it if demand were greater than supply.

Thoughts?

User Avatar
cpdevill307
Wednesday, Jul 17 2024

#feedback The Question Table tab shows both of the following messages:

"Currently showing 4 of 4 completed PrepTests: 158, 156, 155, 154 (tap to edit)"

and

"No LSAT PrepTests scored yet. Score some LSAT PrepTests to see analytics on this page."

PrepTests ·
PT154.S2.Q24
User Avatar
cpdevill307
Tuesday, Jul 16 2024

The problem with C is the phrase "not uncommon" because without a baseline of "uncommon", "not uncommon" does not increase (or decrease) the likelihood that the painter of the battle scene used the painter of the self portrait as a model, which therefore does not weaken it.

The self portrait painter was alive at the time the battle scene was painted, and therefore was able to be a "live model" for the painter of the battle scene, further enhanced by the fact that the self portrait (dated to the same year) resembled the person in the battle scene. It has nothing to do with not respecting the "self" part of the portrait. They could have done the self portrait of themselves and been a live model for the battle scene, there is no inherent contradiction there.

User Avatar
cpdevill307
Monday, Jul 15 2024

9 minutes of prompt analysis only to be immediately told the answer and not be given a chance to apply what we learned. This is by far one of the worst aspects of 7Sage.

User Avatar
cpdevill307
Friday, Jul 12 2024

#feedback Broken record and all, but show all the answer choices before telling us which one is correct. It feels like such a waste of time to listen to 4 minutes of stimulus analysis only to be immediately told which answer is correct without being able to apply what we just listened to.

User Avatar
cpdevill307
Friday, Jul 12 2024

Does E require assuming that there are more than 2 teams? Or is there something in the wording of the question that excludes it, because it can invalidate E as far as I can tell.

If there are 3 teams, E obviously applies. Authors team could have a 34% chance, team B could have 33% chance, team C could have 33% chance (satisfies most likely of a set of events, three events with 34% vs 33% vs 33% chances), then the authors team is not more likely to win than not (because not greater than 50%).

But if there are only 2 teams and the authors team has a 51% chance of winning (satisfies most likely of a set of events, two events with 51% vs 49% chances), then the authors team winning is in fact more likely to win than not (because greater than 50%).

User Avatar
cpdevill307
Thursday, Jul 11 2024

"If you use the technique of kicking the if condition up into the premise set, then (B) should also strike you as completely irrelevant. The kicked-up premise already declared that steel-manufacturing plants can feed heat into the generators."

The analysis above about B is bad. The kicked-up premise does not declare that feeding heat into the generators will produce anything, just that feeding heat into them is possible. So the part of B that connects feeding into the generators with them actually producing electricity is necessary.

Focus on the "using current technology" part being unnecessary.

User Avatar
cpdevill307
Thursday, Jul 11 2024

Is it safe to say that in any NA question if an answer boils down to "an assumption in the argument is true" that that is the correct answer?

Seems stupid to have to ask, but that answer seemed so self evident that it couldn't be correct.

User Avatar
cpdevill307
Thursday, Jul 11 2024

https://7sage.com/lsat_explanations/lsat-64-section-3-question-12/

User Avatar
cpdevill307
Thursday, Jul 11 2024

seemed easier than the first one

User Avatar
cpdevill307
Tuesday, Jul 09 2024

#feedback In the answer B explanation, it is hard to tell how you "fix (B) with the following edit" because we can't see the original answer without scrolling up to the drill results or video. The "Stop reading" instruction is helpful, but not writing out the rest of B makes seeing what you changed with the fix/edit almost impossible.

PrepTests ·
PT145.S4.Q20
User Avatar
cpdevill307
Thursday, Aug 08 2024

#help #feedback How can you just gloss over answer choice A like that with no explanation? If you don't point to one very specific detail of answer choice A it is just as required as answer choice B.

If someone else has been found to have psychic powers and then we find out Selena's claim is true, we aren't determining that psychic powers are possible, because as you said, if someone has psychic powers it is "self evident" that it is possible to have psychic powers. Someone else already was found to have psychic powers so we aren't determining it is possible based on whether Selena's claim is true or not, we are determining nothing.

If someone else has been found to have psychic powers and then we find out Selena's claim is false, we aren't determining that psychic powers are not possible either, because someone else has been found to have psychic powers and again, self evident that they therefore possible. Nothing about this tells us Selena's results are more valid than the others.

So something other than just hand waving makes A wrong.

User Avatar
cpdevill307
Saturday, Jul 06 2024

#feedback C does not "introduce an alternative hypothesis, that it's some third factor", the alternative hypothesis/potential third factor was always there.

C removes 3 alternative hypotheses/third factors (age, weight, and state of health) which strengthens the argument. But the issue is it does not eliminate the possibility of another third factor, so C could still be the cause of both A and B. In absence of E, C would be correct.

E completely removes the possibility that B causes A, so it strengthens it more than partially eliminating C as a factor.

PrepTests ·
PT147.S1.Q11
User Avatar
cpdevill307
Wednesday, Sep 04 2024

Terrible question and even worse "correct" answer, honestly makes me question LSAC as an organization.

C should not be correct because donations are never even mentioned by Nick, just financial support which could simply be buying yearbook ads or buying the naming rights to a stadium or building. Huge assumption there that financial support has to be donations. We all financial support grocery stores by buying groceries, that isn't a donation.

A should be correct because Nick explicitly mentions loyalty being a consideration and Pedro says they should award to the most competitive bid, which obviously excludes picking based on loyalty.

User Avatar
cpdevill307
Wednesday, Jul 03 2024

#feedback I think focusing on the word "only" being too strong in answer A is the better way to eliminate it.

"Many of these hot spots are vulnerable to habitat loss due to commercial development." implies that some hot spots are not vulnerable to habitat loss due to commercial development.

If we aren't bringing in outside knowledge then this is all that we know threatens hot spot habitats. So species in hot spot habitats are not a true subset of "those who are threatened with extinction because of habitat loss" because, again, some are not, the circles merely overlap rather than one being subsumed by the other.

This leads to inefficiency if you concentrate resources on hot spot habitats (answer B) since some don't necessarily need protection, which we trying to avoid due to limited resources. It would be different if answer B read "concentrate their resources on protecting vulnerable hot spot habitats".

This makes answer A sound better because you wont be wasting of the resources on any species that isn't "threatened with extinction", better serving the goal of preserving species.

The problem with answer A is with the word "only" because it might sometimes be a more efficient use of resources to help species threatened with extinction due to reasons other than habitat loss. Stopping drag net fishing in the Caribbean could save more species per resources spent than stopping logging in the Amazon.

Would love some thoughts on this. I might be overthinking the whole thing.

PrepTests ·
PT131.S2.Q24
User Avatar
cpdevill307
Friday, Aug 02 2024

#help I feel like I am missing something that the explanation video is not clarifying. In deciding between answer choices B and D I feel like both require some assumptions and I can't make the distinction to which makes the answer more or less logical.

For B, if 59% of people don't care most about interest rates, but 41% could, and if the 59% were split with 29% caring most about earning miles and 30% caring most about insurance on their purchases, then 41% caring most about interest rates would be the thing that that most consumers care the most about, making B incorrect. Obviously the various splits could go a lot of ways, but valid assumptions can be made to make B logical or not.

For D, there is nothing to suggest that most consumers (59%) would care about the length of time a company allows to pay the balance due before interest accrues unless it were something we assume that something unreasonable like less than a day is possible or unless we assume that paying the balance requires writing a check (something we are explicitly told the 59% are avoiding). This question is older, but not so old that I think that is a reasonable assumption, maybe auto-payments/paying online wasn't as common as I am remembering in 2009.

OR is everything I wrote out above irrelevant because the prompt says "credit card companies tend to..." meaning "most", and D does not say "most"? Meaning D makes a more egregious logical error in implying "all" rather than "most" than B makes in glossing over situations in which in which it isn't true?

User Avatar
cpdevill307
Tuesday, Jul 02 2024

What would the contrapositive of "eat-hamburger and /drink-beer" be? Is it even possible to take the contrapositive of that?

Confirm action

Are you sure?